Partial Differential Equation using separation of variables

Click For Summary
The discussion focuses on solving a heat flow problem using separation of variables for a specific partial differential equation. The user has successfully separated variables and applied boundary conditions but is struggling with the piecewise initial conditions. To address this, it is suggested to find the Fourier cosine coefficients by integrating over the defined intervals corresponding to the piecewise function. The integration should be performed from 0 to L/4, L/4 to 3L/4, and 3L/4 to L, but effectively only the interval from L/4 to 3L/4 contributes since the function is zero outside this range. Properly applying these integrals will allow for the correct application of the initial conditions.
Hendrick
Messages
41
Reaction score
0

Homework Statement


Solve the heat flow problem using the method of separation of variables:


Homework Equations


PDE:\frac{\partial u}{\partial t}=k\frac{\partial^{2} u}{\partial t^{2}}
for 0<x<L, 0<t<\infty

BC's:\frac{\partial u}{\partial x}(0,t)=0,\frac{\partial u}{\partial x}(L,t)=0
for 0<t<\infty

IC's: u(x,0)=
{0, 0&lt;x&lt;L/4
{1, L/4&lt;x&lt;3L/4
{0, 3L/4&lt;x&lt;L
(Piecewise IC)

The Attempt at a Solution


I have separated the variables, then applied the boundary conditions. I am stuck on applying the initial conditions.

I have come up with a general product solution of u_{n}=F_{n}cos(\frac{n \pi x}{L}) e^{-k(\frac{n \pi x}{L})^{2}t}

Trying to combine all product solutions and match the initial data:
u(x,0)=f(x)
\sum^{\infty}_{n=1}u_{n}(x,0)=f(x)
\sum^{\infty}_{n=1}F_{n}cos(\frac{n \pi x}{L})=f(x)


I don't know how to apply the piecewise initial condition, any help would be appreciated. Thank you
 
Physics news on Phys.org
Once you have
\sum^{\infty}_{n=1}F_{n}cos(\frac{n \pi x}{L})=f(x)
you do it exactly the same way you would if f were not "piecewise". Find the Fourier cosine coefficients by doing the appropriate integrals for C0 and Cn for n> 0. The only difference "piecewise" makes is that instead of integrating a single formula from 0 to L, you integrate using the given formulas from 0 to L/4, L/4 to 3L/4, 3L/4 to L and adding those integrals. (Which I notice now is just integrating from L/4 to 3L/4 since outside that the function is 0. The difference between this and just doing the problem on the interval from L/4 to 3L/4 is that you use the whole interval, of length L in determining the "normalization" constant.)
 
Question: A clock's minute hand has length 4 and its hour hand has length 3. What is the distance between the tips at the moment when it is increasing most rapidly?(Putnam Exam Question) Answer: Making assumption that both the hands moves at constant angular velocities, the answer is ## \sqrt{7} .## But don't you think this assumption is somewhat doubtful and wrong?

Similar threads

Replies
1
Views
2K
  • · Replies 1 ·
Replies
1
Views
1K
  • · Replies 11 ·
Replies
11
Views
2K
Replies
4
Views
1K
  • · Replies 6 ·
Replies
6
Views
2K
  • · Replies 11 ·
Replies
11
Views
3K
  • · Replies 4 ·
Replies
4
Views
1K
Replies
5
Views
2K
  • · Replies 2 ·
Replies
2
Views
2K
Replies
9
Views
2K